Showing that $int_{0}^{infty} u^{-alpha} sin(u) , du >0$ for $0<alpha<1$












0












$begingroup$


Does anyone know how to show $int_{0}^{infty} u^{-alpha} sin(u) , du >0$ for $0<alpha<1$ (without explicitly having to calculate the exact value)?










share|cite|improve this question









$endgroup$

















    0












    $begingroup$


    Does anyone know how to show $int_{0}^{infty} u^{-alpha} sin(u) , du >0$ for $0<alpha<1$ (without explicitly having to calculate the exact value)?










    share|cite|improve this question









    $endgroup$















      0












      0








      0





      $begingroup$


      Does anyone know how to show $int_{0}^{infty} u^{-alpha} sin(u) , du >0$ for $0<alpha<1$ (without explicitly having to calculate the exact value)?










      share|cite|improve this question









      $endgroup$




      Does anyone know how to show $int_{0}^{infty} u^{-alpha} sin(u) , du >0$ for $0<alpha<1$ (without explicitly having to calculate the exact value)?







      real-analysis improper-integrals






      share|cite|improve this question













      share|cite|improve this question











      share|cite|improve this question




      share|cite|improve this question










      asked Dec 10 '18 at 18:51









      VercingetorixVercingetorix

      285




      285






















          3 Answers
          3






          active

          oldest

          votes


















          3












          $begingroup$

          For another way, integrate by parts:
          begin{align*}I(alpha,M) &= int_0^M u^{-alpha}sin u,du\
          &= left[u^{-alpha}cdot (1-cos u)right]_{u=0}^{u=M} + int_0^M alpha u^{-alpha-1}cdot (1-cos u),du\
          I(alpha,M) &= M^{-u}cdot (1-cos M) + alphaint_0^M u^{-alpha-1}cdot (1-cos u),duend{align*}

          For positive $alpha$ and $M$, both terms are clearly nonnegative; the first is the product of two nonnegative terms, and the second is the integral of a nonnegative function. Evaluating that first term to be zero at zero - well, actually, that's a limit, and it requires $u<2$ so that the zero of the $1-cos$ overwhelms the growth of the $u^{-alpha}$.

          As $Mtoinfty$, this calculation doubles as a proof that this improper integral converges. For $0<alphale 1$, we've converted a conditionally convergent integral into an absolutely convergent one with this manipulation; the new integral converges as $Mtoinfty$ by comparison with $int 2u^{-alpha -1}$ and the boundary term goes to zero.



          I'd also like to call out two choices that may seem odd at first but turn out to be broadly useful techniques:

          - "Integrate the part that wiggles". A lot of the time, when we integrate by parts, we're not actually trying to find an antiderivative. With the integral of an oscillating function times a decaying function, one common option is to integrate by parts to improve convergence behavior, integrating the oscillating part and differentiating whatever's left over. If we have to introduce some factor here to make the oscillating part's integral work, we do it.

          - Choosing $1-cos$ as the antiderivative instead of $cos$. That $+C$ in the indefinite integral gets ignored a lot, but it's a free choice; we should use whatever works for us, not just whatever looks like zero. Here, since we care about the integral being positive, we choose $1-cos$ to make that positive after integration, and also to zero out the boundary term at zero. Actually, for the range of $alpha$ we care about, any other choice of antiderivative would have split the integral as the sum of a divergent limit and a divergent (at zero) improper integral. That's definitely not a desirable outcome.






          share|cite|improve this answer









          $endgroup$













          • $begingroup$
            $int 2u^{-alpha-1}$ does not converge
            $endgroup$
            – Vercingetorix
            Dec 12 '18 at 0:03










          • $begingroup$
            $int^{infty} 2u^{-alpha-1}$ does converge, which is what I was thinking of there. On the $int_0$ side, we have the $1-cos u$ factor to get us an extra $u^2$ factor, so it's not even improper there.
            $endgroup$
            – jmerry
            Dec 12 '18 at 0:45



















          2












          $begingroup$

          Since the Laplace transform is a self-adjoint operator, the Laplace transform of $sin(x)$ is $frac{1}{s^2+1}$ and the inverse Laplace transform of $u^{-alpha}$ is $frac{s^{alpha-1}}{Gamma(alpha)}$,
          $$ int_{0}^{+infty} u^{-alpha}sin(u),du = frac{1}{Gamma(alpha)}int_{0}^{+infty}frac{s^{alpha-1}}{s^2+1},ds=cosleft(frac{pialpha}{2}right)Gamma(1-alpha)$$
          for any $alphain(0,1)$. On the other hand the finite-ness of the LHS just follows from integration by parts:
          $$ int_{0}^{+infty} u^{-alpha}sin(u),du = alphaint_{0}^{+infty}frac{1-cos u}{u^{alpha+1}},du$$
          since $1-cos u$ behaves like $frac{u^2}{2}$ in a right neighbourhood of the origin and it is non-negative and bounded far from the origin.






          share|cite|improve this answer









          $endgroup$













          • $begingroup$
            I guess this doesn't really fall in the category "without explicitly having to calculate the exact value", but +1 anyway because I always learn from your integrations.
            $endgroup$
            – Federico
            Dec 10 '18 at 19:25










          • $begingroup$
            @Federico: the two parts are independent: you may prove the finite-ness through the closed form, or just invoke integration by parts.
            $endgroup$
            – Jack D'Aurizio
            Dec 10 '18 at 19:27



















          0












          $begingroup$

          $$ int_0^infty u^{-alpha} sin(u); du = sum_{n=0}^infty
          int_0^{pi} left((2n pi + t)^{-alpha} - ((2n+1)pi + t)^{-alpha}right) sin(t); dt $$



          and the integrand is positive.






          share|cite|improve this answer









          $endgroup$













          • $begingroup$
            I see this is clear if we can interchange the integration and summation, but why can we do that?. Also why is the integrand positive?
            $endgroup$
            – Vercingetorix
            Dec 10 '18 at 19:19










          • $begingroup$
            $int_0^R = sum_{n=0}^{lfloor R/(2pi) rfloor} int_{2pi n}^{2pi (n+1)} + ldots$
            $endgroup$
            – Robert Israel
            Dec 10 '18 at 20:50











          Your Answer





          StackExchange.ifUsing("editor", function () {
          return StackExchange.using("mathjaxEditing", function () {
          StackExchange.MarkdownEditor.creationCallbacks.add(function (editor, postfix) {
          StackExchange.mathjaxEditing.prepareWmdForMathJax(editor, postfix, [["$", "$"], ["\\(","\\)"]]);
          });
          });
          }, "mathjax-editing");

          StackExchange.ready(function() {
          var channelOptions = {
          tags: "".split(" "),
          id: "69"
          };
          initTagRenderer("".split(" "), "".split(" "), channelOptions);

          StackExchange.using("externalEditor", function() {
          // Have to fire editor after snippets, if snippets enabled
          if (StackExchange.settings.snippets.snippetsEnabled) {
          StackExchange.using("snippets", function() {
          createEditor();
          });
          }
          else {
          createEditor();
          }
          });

          function createEditor() {
          StackExchange.prepareEditor({
          heartbeatType: 'answer',
          autoActivateHeartbeat: false,
          convertImagesToLinks: true,
          noModals: true,
          showLowRepImageUploadWarning: true,
          reputationToPostImages: 10,
          bindNavPrevention: true,
          postfix: "",
          imageUploader: {
          brandingHtml: "Powered by u003ca class="icon-imgur-white" href="https://imgur.com/"u003eu003c/au003e",
          contentPolicyHtml: "User contributions licensed under u003ca href="https://creativecommons.org/licenses/by-sa/3.0/"u003ecc by-sa 3.0 with attribution requiredu003c/au003e u003ca href="https://stackoverflow.com/legal/content-policy"u003e(content policy)u003c/au003e",
          allowUrls: true
          },
          noCode: true, onDemand: true,
          discardSelector: ".discard-answer"
          ,immediatelyShowMarkdownHelp:true
          });


          }
          });














          draft saved

          draft discarded


















          StackExchange.ready(
          function () {
          StackExchange.openid.initPostLogin('.new-post-login', 'https%3a%2f%2fmath.stackexchange.com%2fquestions%2f3034331%2fshowing-that-int-0-infty-u-alpha-sinu-du-0-for-0-alpha1%23new-answer', 'question_page');
          }
          );

          Post as a guest















          Required, but never shown

























          3 Answers
          3






          active

          oldest

          votes








          3 Answers
          3






          active

          oldest

          votes









          active

          oldest

          votes






          active

          oldest

          votes









          3












          $begingroup$

          For another way, integrate by parts:
          begin{align*}I(alpha,M) &= int_0^M u^{-alpha}sin u,du\
          &= left[u^{-alpha}cdot (1-cos u)right]_{u=0}^{u=M} + int_0^M alpha u^{-alpha-1}cdot (1-cos u),du\
          I(alpha,M) &= M^{-u}cdot (1-cos M) + alphaint_0^M u^{-alpha-1}cdot (1-cos u),duend{align*}

          For positive $alpha$ and $M$, both terms are clearly nonnegative; the first is the product of two nonnegative terms, and the second is the integral of a nonnegative function. Evaluating that first term to be zero at zero - well, actually, that's a limit, and it requires $u<2$ so that the zero of the $1-cos$ overwhelms the growth of the $u^{-alpha}$.

          As $Mtoinfty$, this calculation doubles as a proof that this improper integral converges. For $0<alphale 1$, we've converted a conditionally convergent integral into an absolutely convergent one with this manipulation; the new integral converges as $Mtoinfty$ by comparison with $int 2u^{-alpha -1}$ and the boundary term goes to zero.



          I'd also like to call out two choices that may seem odd at first but turn out to be broadly useful techniques:

          - "Integrate the part that wiggles". A lot of the time, when we integrate by parts, we're not actually trying to find an antiderivative. With the integral of an oscillating function times a decaying function, one common option is to integrate by parts to improve convergence behavior, integrating the oscillating part and differentiating whatever's left over. If we have to introduce some factor here to make the oscillating part's integral work, we do it.

          - Choosing $1-cos$ as the antiderivative instead of $cos$. That $+C$ in the indefinite integral gets ignored a lot, but it's a free choice; we should use whatever works for us, not just whatever looks like zero. Here, since we care about the integral being positive, we choose $1-cos$ to make that positive after integration, and also to zero out the boundary term at zero. Actually, for the range of $alpha$ we care about, any other choice of antiderivative would have split the integral as the sum of a divergent limit and a divergent (at zero) improper integral. That's definitely not a desirable outcome.






          share|cite|improve this answer









          $endgroup$













          • $begingroup$
            $int 2u^{-alpha-1}$ does not converge
            $endgroup$
            – Vercingetorix
            Dec 12 '18 at 0:03










          • $begingroup$
            $int^{infty} 2u^{-alpha-1}$ does converge, which is what I was thinking of there. On the $int_0$ side, we have the $1-cos u$ factor to get us an extra $u^2$ factor, so it's not even improper there.
            $endgroup$
            – jmerry
            Dec 12 '18 at 0:45
















          3












          $begingroup$

          For another way, integrate by parts:
          begin{align*}I(alpha,M) &= int_0^M u^{-alpha}sin u,du\
          &= left[u^{-alpha}cdot (1-cos u)right]_{u=0}^{u=M} + int_0^M alpha u^{-alpha-1}cdot (1-cos u),du\
          I(alpha,M) &= M^{-u}cdot (1-cos M) + alphaint_0^M u^{-alpha-1}cdot (1-cos u),duend{align*}

          For positive $alpha$ and $M$, both terms are clearly nonnegative; the first is the product of two nonnegative terms, and the second is the integral of a nonnegative function. Evaluating that first term to be zero at zero - well, actually, that's a limit, and it requires $u<2$ so that the zero of the $1-cos$ overwhelms the growth of the $u^{-alpha}$.

          As $Mtoinfty$, this calculation doubles as a proof that this improper integral converges. For $0<alphale 1$, we've converted a conditionally convergent integral into an absolutely convergent one with this manipulation; the new integral converges as $Mtoinfty$ by comparison with $int 2u^{-alpha -1}$ and the boundary term goes to zero.



          I'd also like to call out two choices that may seem odd at first but turn out to be broadly useful techniques:

          - "Integrate the part that wiggles". A lot of the time, when we integrate by parts, we're not actually trying to find an antiderivative. With the integral of an oscillating function times a decaying function, one common option is to integrate by parts to improve convergence behavior, integrating the oscillating part and differentiating whatever's left over. If we have to introduce some factor here to make the oscillating part's integral work, we do it.

          - Choosing $1-cos$ as the antiderivative instead of $cos$. That $+C$ in the indefinite integral gets ignored a lot, but it's a free choice; we should use whatever works for us, not just whatever looks like zero. Here, since we care about the integral being positive, we choose $1-cos$ to make that positive after integration, and also to zero out the boundary term at zero. Actually, for the range of $alpha$ we care about, any other choice of antiderivative would have split the integral as the sum of a divergent limit and a divergent (at zero) improper integral. That's definitely not a desirable outcome.






          share|cite|improve this answer









          $endgroup$













          • $begingroup$
            $int 2u^{-alpha-1}$ does not converge
            $endgroup$
            – Vercingetorix
            Dec 12 '18 at 0:03










          • $begingroup$
            $int^{infty} 2u^{-alpha-1}$ does converge, which is what I was thinking of there. On the $int_0$ side, we have the $1-cos u$ factor to get us an extra $u^2$ factor, so it's not even improper there.
            $endgroup$
            – jmerry
            Dec 12 '18 at 0:45














          3












          3








          3





          $begingroup$

          For another way, integrate by parts:
          begin{align*}I(alpha,M) &= int_0^M u^{-alpha}sin u,du\
          &= left[u^{-alpha}cdot (1-cos u)right]_{u=0}^{u=M} + int_0^M alpha u^{-alpha-1}cdot (1-cos u),du\
          I(alpha,M) &= M^{-u}cdot (1-cos M) + alphaint_0^M u^{-alpha-1}cdot (1-cos u),duend{align*}

          For positive $alpha$ and $M$, both terms are clearly nonnegative; the first is the product of two nonnegative terms, and the second is the integral of a nonnegative function. Evaluating that first term to be zero at zero - well, actually, that's a limit, and it requires $u<2$ so that the zero of the $1-cos$ overwhelms the growth of the $u^{-alpha}$.

          As $Mtoinfty$, this calculation doubles as a proof that this improper integral converges. For $0<alphale 1$, we've converted a conditionally convergent integral into an absolutely convergent one with this manipulation; the new integral converges as $Mtoinfty$ by comparison with $int 2u^{-alpha -1}$ and the boundary term goes to zero.



          I'd also like to call out two choices that may seem odd at first but turn out to be broadly useful techniques:

          - "Integrate the part that wiggles". A lot of the time, when we integrate by parts, we're not actually trying to find an antiderivative. With the integral of an oscillating function times a decaying function, one common option is to integrate by parts to improve convergence behavior, integrating the oscillating part and differentiating whatever's left over. If we have to introduce some factor here to make the oscillating part's integral work, we do it.

          - Choosing $1-cos$ as the antiderivative instead of $cos$. That $+C$ in the indefinite integral gets ignored a lot, but it's a free choice; we should use whatever works for us, not just whatever looks like zero. Here, since we care about the integral being positive, we choose $1-cos$ to make that positive after integration, and also to zero out the boundary term at zero. Actually, for the range of $alpha$ we care about, any other choice of antiderivative would have split the integral as the sum of a divergent limit and a divergent (at zero) improper integral. That's definitely not a desirable outcome.






          share|cite|improve this answer









          $endgroup$



          For another way, integrate by parts:
          begin{align*}I(alpha,M) &= int_0^M u^{-alpha}sin u,du\
          &= left[u^{-alpha}cdot (1-cos u)right]_{u=0}^{u=M} + int_0^M alpha u^{-alpha-1}cdot (1-cos u),du\
          I(alpha,M) &= M^{-u}cdot (1-cos M) + alphaint_0^M u^{-alpha-1}cdot (1-cos u),duend{align*}

          For positive $alpha$ and $M$, both terms are clearly nonnegative; the first is the product of two nonnegative terms, and the second is the integral of a nonnegative function. Evaluating that first term to be zero at zero - well, actually, that's a limit, and it requires $u<2$ so that the zero of the $1-cos$ overwhelms the growth of the $u^{-alpha}$.

          As $Mtoinfty$, this calculation doubles as a proof that this improper integral converges. For $0<alphale 1$, we've converted a conditionally convergent integral into an absolutely convergent one with this manipulation; the new integral converges as $Mtoinfty$ by comparison with $int 2u^{-alpha -1}$ and the boundary term goes to zero.



          I'd also like to call out two choices that may seem odd at first but turn out to be broadly useful techniques:

          - "Integrate the part that wiggles". A lot of the time, when we integrate by parts, we're not actually trying to find an antiderivative. With the integral of an oscillating function times a decaying function, one common option is to integrate by parts to improve convergence behavior, integrating the oscillating part and differentiating whatever's left over. If we have to introduce some factor here to make the oscillating part's integral work, we do it.

          - Choosing $1-cos$ as the antiderivative instead of $cos$. That $+C$ in the indefinite integral gets ignored a lot, but it's a free choice; we should use whatever works for us, not just whatever looks like zero. Here, since we care about the integral being positive, we choose $1-cos$ to make that positive after integration, and also to zero out the boundary term at zero. Actually, for the range of $alpha$ we care about, any other choice of antiderivative would have split the integral as the sum of a divergent limit and a divergent (at zero) improper integral. That's definitely not a desirable outcome.







          share|cite|improve this answer












          share|cite|improve this answer



          share|cite|improve this answer










          answered Dec 10 '18 at 19:50









          jmerryjmerry

          9,7581225




          9,7581225












          • $begingroup$
            $int 2u^{-alpha-1}$ does not converge
            $endgroup$
            – Vercingetorix
            Dec 12 '18 at 0:03










          • $begingroup$
            $int^{infty} 2u^{-alpha-1}$ does converge, which is what I was thinking of there. On the $int_0$ side, we have the $1-cos u$ factor to get us an extra $u^2$ factor, so it's not even improper there.
            $endgroup$
            – jmerry
            Dec 12 '18 at 0:45


















          • $begingroup$
            $int 2u^{-alpha-1}$ does not converge
            $endgroup$
            – Vercingetorix
            Dec 12 '18 at 0:03










          • $begingroup$
            $int^{infty} 2u^{-alpha-1}$ does converge, which is what I was thinking of there. On the $int_0$ side, we have the $1-cos u$ factor to get us an extra $u^2$ factor, so it's not even improper there.
            $endgroup$
            – jmerry
            Dec 12 '18 at 0:45
















          $begingroup$
          $int 2u^{-alpha-1}$ does not converge
          $endgroup$
          – Vercingetorix
          Dec 12 '18 at 0:03




          $begingroup$
          $int 2u^{-alpha-1}$ does not converge
          $endgroup$
          – Vercingetorix
          Dec 12 '18 at 0:03












          $begingroup$
          $int^{infty} 2u^{-alpha-1}$ does converge, which is what I was thinking of there. On the $int_0$ side, we have the $1-cos u$ factor to get us an extra $u^2$ factor, so it's not even improper there.
          $endgroup$
          – jmerry
          Dec 12 '18 at 0:45




          $begingroup$
          $int^{infty} 2u^{-alpha-1}$ does converge, which is what I was thinking of there. On the $int_0$ side, we have the $1-cos u$ factor to get us an extra $u^2$ factor, so it's not even improper there.
          $endgroup$
          – jmerry
          Dec 12 '18 at 0:45











          2












          $begingroup$

          Since the Laplace transform is a self-adjoint operator, the Laplace transform of $sin(x)$ is $frac{1}{s^2+1}$ and the inverse Laplace transform of $u^{-alpha}$ is $frac{s^{alpha-1}}{Gamma(alpha)}$,
          $$ int_{0}^{+infty} u^{-alpha}sin(u),du = frac{1}{Gamma(alpha)}int_{0}^{+infty}frac{s^{alpha-1}}{s^2+1},ds=cosleft(frac{pialpha}{2}right)Gamma(1-alpha)$$
          for any $alphain(0,1)$. On the other hand the finite-ness of the LHS just follows from integration by parts:
          $$ int_{0}^{+infty} u^{-alpha}sin(u),du = alphaint_{0}^{+infty}frac{1-cos u}{u^{alpha+1}},du$$
          since $1-cos u$ behaves like $frac{u^2}{2}$ in a right neighbourhood of the origin and it is non-negative and bounded far from the origin.






          share|cite|improve this answer









          $endgroup$













          • $begingroup$
            I guess this doesn't really fall in the category "without explicitly having to calculate the exact value", but +1 anyway because I always learn from your integrations.
            $endgroup$
            – Federico
            Dec 10 '18 at 19:25










          • $begingroup$
            @Federico: the two parts are independent: you may prove the finite-ness through the closed form, or just invoke integration by parts.
            $endgroup$
            – Jack D'Aurizio
            Dec 10 '18 at 19:27
















          2












          $begingroup$

          Since the Laplace transform is a self-adjoint operator, the Laplace transform of $sin(x)$ is $frac{1}{s^2+1}$ and the inverse Laplace transform of $u^{-alpha}$ is $frac{s^{alpha-1}}{Gamma(alpha)}$,
          $$ int_{0}^{+infty} u^{-alpha}sin(u),du = frac{1}{Gamma(alpha)}int_{0}^{+infty}frac{s^{alpha-1}}{s^2+1},ds=cosleft(frac{pialpha}{2}right)Gamma(1-alpha)$$
          for any $alphain(0,1)$. On the other hand the finite-ness of the LHS just follows from integration by parts:
          $$ int_{0}^{+infty} u^{-alpha}sin(u),du = alphaint_{0}^{+infty}frac{1-cos u}{u^{alpha+1}},du$$
          since $1-cos u$ behaves like $frac{u^2}{2}$ in a right neighbourhood of the origin and it is non-negative and bounded far from the origin.






          share|cite|improve this answer









          $endgroup$













          • $begingroup$
            I guess this doesn't really fall in the category "without explicitly having to calculate the exact value", but +1 anyway because I always learn from your integrations.
            $endgroup$
            – Federico
            Dec 10 '18 at 19:25










          • $begingroup$
            @Federico: the two parts are independent: you may prove the finite-ness through the closed form, or just invoke integration by parts.
            $endgroup$
            – Jack D'Aurizio
            Dec 10 '18 at 19:27














          2












          2








          2





          $begingroup$

          Since the Laplace transform is a self-adjoint operator, the Laplace transform of $sin(x)$ is $frac{1}{s^2+1}$ and the inverse Laplace transform of $u^{-alpha}$ is $frac{s^{alpha-1}}{Gamma(alpha)}$,
          $$ int_{0}^{+infty} u^{-alpha}sin(u),du = frac{1}{Gamma(alpha)}int_{0}^{+infty}frac{s^{alpha-1}}{s^2+1},ds=cosleft(frac{pialpha}{2}right)Gamma(1-alpha)$$
          for any $alphain(0,1)$. On the other hand the finite-ness of the LHS just follows from integration by parts:
          $$ int_{0}^{+infty} u^{-alpha}sin(u),du = alphaint_{0}^{+infty}frac{1-cos u}{u^{alpha+1}},du$$
          since $1-cos u$ behaves like $frac{u^2}{2}$ in a right neighbourhood of the origin and it is non-negative and bounded far from the origin.






          share|cite|improve this answer









          $endgroup$



          Since the Laplace transform is a self-adjoint operator, the Laplace transform of $sin(x)$ is $frac{1}{s^2+1}$ and the inverse Laplace transform of $u^{-alpha}$ is $frac{s^{alpha-1}}{Gamma(alpha)}$,
          $$ int_{0}^{+infty} u^{-alpha}sin(u),du = frac{1}{Gamma(alpha)}int_{0}^{+infty}frac{s^{alpha-1}}{s^2+1},ds=cosleft(frac{pialpha}{2}right)Gamma(1-alpha)$$
          for any $alphain(0,1)$. On the other hand the finite-ness of the LHS just follows from integration by parts:
          $$ int_{0}^{+infty} u^{-alpha}sin(u),du = alphaint_{0}^{+infty}frac{1-cos u}{u^{alpha+1}},du$$
          since $1-cos u$ behaves like $frac{u^2}{2}$ in a right neighbourhood of the origin and it is non-negative and bounded far from the origin.







          share|cite|improve this answer












          share|cite|improve this answer



          share|cite|improve this answer










          answered Dec 10 '18 at 19:15









          Jack D'AurizioJack D'Aurizio

          290k33282662




          290k33282662












          • $begingroup$
            I guess this doesn't really fall in the category "without explicitly having to calculate the exact value", but +1 anyway because I always learn from your integrations.
            $endgroup$
            – Federico
            Dec 10 '18 at 19:25










          • $begingroup$
            @Federico: the two parts are independent: you may prove the finite-ness through the closed form, or just invoke integration by parts.
            $endgroup$
            – Jack D'Aurizio
            Dec 10 '18 at 19:27


















          • $begingroup$
            I guess this doesn't really fall in the category "without explicitly having to calculate the exact value", but +1 anyway because I always learn from your integrations.
            $endgroup$
            – Federico
            Dec 10 '18 at 19:25










          • $begingroup$
            @Federico: the two parts are independent: you may prove the finite-ness through the closed form, or just invoke integration by parts.
            $endgroup$
            – Jack D'Aurizio
            Dec 10 '18 at 19:27
















          $begingroup$
          I guess this doesn't really fall in the category "without explicitly having to calculate the exact value", but +1 anyway because I always learn from your integrations.
          $endgroup$
          – Federico
          Dec 10 '18 at 19:25




          $begingroup$
          I guess this doesn't really fall in the category "without explicitly having to calculate the exact value", but +1 anyway because I always learn from your integrations.
          $endgroup$
          – Federico
          Dec 10 '18 at 19:25












          $begingroup$
          @Federico: the two parts are independent: you may prove the finite-ness through the closed form, or just invoke integration by parts.
          $endgroup$
          – Jack D'Aurizio
          Dec 10 '18 at 19:27




          $begingroup$
          @Federico: the two parts are independent: you may prove the finite-ness through the closed form, or just invoke integration by parts.
          $endgroup$
          – Jack D'Aurizio
          Dec 10 '18 at 19:27











          0












          $begingroup$

          $$ int_0^infty u^{-alpha} sin(u); du = sum_{n=0}^infty
          int_0^{pi} left((2n pi + t)^{-alpha} - ((2n+1)pi + t)^{-alpha}right) sin(t); dt $$



          and the integrand is positive.






          share|cite|improve this answer









          $endgroup$













          • $begingroup$
            I see this is clear if we can interchange the integration and summation, but why can we do that?. Also why is the integrand positive?
            $endgroup$
            – Vercingetorix
            Dec 10 '18 at 19:19










          • $begingroup$
            $int_0^R = sum_{n=0}^{lfloor R/(2pi) rfloor} int_{2pi n}^{2pi (n+1)} + ldots$
            $endgroup$
            – Robert Israel
            Dec 10 '18 at 20:50
















          0












          $begingroup$

          $$ int_0^infty u^{-alpha} sin(u); du = sum_{n=0}^infty
          int_0^{pi} left((2n pi + t)^{-alpha} - ((2n+1)pi + t)^{-alpha}right) sin(t); dt $$



          and the integrand is positive.






          share|cite|improve this answer









          $endgroup$













          • $begingroup$
            I see this is clear if we can interchange the integration and summation, but why can we do that?. Also why is the integrand positive?
            $endgroup$
            – Vercingetorix
            Dec 10 '18 at 19:19










          • $begingroup$
            $int_0^R = sum_{n=0}^{lfloor R/(2pi) rfloor} int_{2pi n}^{2pi (n+1)} + ldots$
            $endgroup$
            – Robert Israel
            Dec 10 '18 at 20:50














          0












          0








          0





          $begingroup$

          $$ int_0^infty u^{-alpha} sin(u); du = sum_{n=0}^infty
          int_0^{pi} left((2n pi + t)^{-alpha} - ((2n+1)pi + t)^{-alpha}right) sin(t); dt $$



          and the integrand is positive.






          share|cite|improve this answer









          $endgroup$



          $$ int_0^infty u^{-alpha} sin(u); du = sum_{n=0}^infty
          int_0^{pi} left((2n pi + t)^{-alpha} - ((2n+1)pi + t)^{-alpha}right) sin(t); dt $$



          and the integrand is positive.







          share|cite|improve this answer












          share|cite|improve this answer



          share|cite|improve this answer










          answered Dec 10 '18 at 19:02









          Robert IsraelRobert Israel

          324k23214468




          324k23214468












          • $begingroup$
            I see this is clear if we can interchange the integration and summation, but why can we do that?. Also why is the integrand positive?
            $endgroup$
            – Vercingetorix
            Dec 10 '18 at 19:19










          • $begingroup$
            $int_0^R = sum_{n=0}^{lfloor R/(2pi) rfloor} int_{2pi n}^{2pi (n+1)} + ldots$
            $endgroup$
            – Robert Israel
            Dec 10 '18 at 20:50


















          • $begingroup$
            I see this is clear if we can interchange the integration and summation, but why can we do that?. Also why is the integrand positive?
            $endgroup$
            – Vercingetorix
            Dec 10 '18 at 19:19










          • $begingroup$
            $int_0^R = sum_{n=0}^{lfloor R/(2pi) rfloor} int_{2pi n}^{2pi (n+1)} + ldots$
            $endgroup$
            – Robert Israel
            Dec 10 '18 at 20:50
















          $begingroup$
          I see this is clear if we can interchange the integration and summation, but why can we do that?. Also why is the integrand positive?
          $endgroup$
          – Vercingetorix
          Dec 10 '18 at 19:19




          $begingroup$
          I see this is clear if we can interchange the integration and summation, but why can we do that?. Also why is the integrand positive?
          $endgroup$
          – Vercingetorix
          Dec 10 '18 at 19:19












          $begingroup$
          $int_0^R = sum_{n=0}^{lfloor R/(2pi) rfloor} int_{2pi n}^{2pi (n+1)} + ldots$
          $endgroup$
          – Robert Israel
          Dec 10 '18 at 20:50




          $begingroup$
          $int_0^R = sum_{n=0}^{lfloor R/(2pi) rfloor} int_{2pi n}^{2pi (n+1)} + ldots$
          $endgroup$
          – Robert Israel
          Dec 10 '18 at 20:50


















          draft saved

          draft discarded




















































          Thanks for contributing an answer to Mathematics Stack Exchange!


          • Please be sure to answer the question. Provide details and share your research!

          But avoid



          • Asking for help, clarification, or responding to other answers.

          • Making statements based on opinion; back them up with references or personal experience.


          Use MathJax to format equations. MathJax reference.


          To learn more, see our tips on writing great answers.




          draft saved


          draft discarded














          StackExchange.ready(
          function () {
          StackExchange.openid.initPostLogin('.new-post-login', 'https%3a%2f%2fmath.stackexchange.com%2fquestions%2f3034331%2fshowing-that-int-0-infty-u-alpha-sinu-du-0-for-0-alpha1%23new-answer', 'question_page');
          }
          );

          Post as a guest















          Required, but never shown





















































          Required, but never shown














          Required, but never shown












          Required, but never shown







          Required, but never shown

































          Required, but never shown














          Required, but never shown












          Required, but never shown







          Required, but never shown







          Popular posts from this blog

          Plaza Victoria

          In PowerPoint, is there a keyboard shortcut for bulleted / numbered list?

          How to put 3 figures in Latex with 2 figures side by side and 1 below these side by side images but in...